Fourier Transform: Limit in Infinity of Exponential Function

Click For Summary
The discussion focuses on the limit of an exponential function with an imaginary exponent in the context of Fourier transforms. It clarifies that the expression e^(ikx) is bounded due to the properties of sine and cosine, while e^(-ax) approaches zero as x tends to infinity. Consequently, the product of these two terms results in the limit approaching zero. The confusion arises from the expectation that the oscillatory nature of sine and cosine would not lead to a zero limit. Ultimately, the limit of e^(ik-a)x as x approaches infinity is confirmed to be zero.
aaaa202
Messages
1,144
Reaction score
2
In calculating some basic Fourier transform I seem stumble on the proble that I don't know how to take the limit in infinity of an exponentialfunction with imaginary exponent. In the attached example it just seems to give zero but I don't know what asserts this property. I would have thought that it would yield something infinite since a cosine or sine does not go to zero at infinity. What is done to arrive at the attached result?
 

Attachments

  • fourier transform.png
    fourier transform.png
    5.6 KB · Views: 453
Physics news on Phys.org
aaaa202 said:
In calculating some basic Fourier transform I seem stumble on the proble that I don't know how to take the limit in infinity of an exponentialfunction with imaginary exponent. In the attached example it just seems to give zero but I don't know what asserts this property. I would have thought that it would yield something infinite since a cosine or sine does not go to zero at infinity. What is done to arrive at the attached result?

Think of ##e^{(ik-a)x}## as equal to ##e^{ikx} \cdot e^{-ax}##. On the right hand side while ##e^{ikx}## is not really defined for when x tends to infinity, it is still bounded (because sine and cosine are bounded) and the term ##e^{-ax}## does tend to 0. So that the product tends to 0 and you get ##\lim _{x \to \infty}e^{(ik-a)x} =0##.
 
Question: A clock's minute hand has length 4 and its hour hand has length 3. What is the distance between the tips at the moment when it is increasing most rapidly?(Putnam Exam Question) Answer: Making assumption that both the hands moves at constant angular velocities, the answer is ## \sqrt{7} .## But don't you think this assumption is somewhat doubtful and wrong?

Similar threads

  • · Replies 5 ·
Replies
5
Views
2K
  • · Replies 2 ·
Replies
2
Views
3K
Replies
2
Views
2K
  • · Replies 2 ·
Replies
2
Views
2K
  • · Replies 1 ·
Replies
1
Views
2K
  • · Replies 6 ·
Replies
6
Views
2K
Replies
8
Views
2K
  • · Replies 6 ·
Replies
6
Views
3K
  • · Replies 6 ·
Replies
6
Views
2K
  • · Replies 9 ·
Replies
9
Views
2K